You are on page 1of 85
1. A physical therapist is treating a child with spastic cerebral palsy who is 3 years old cognitively but at a 6-month-old gross developmental level. What is an appropriate treatment activity for this child? A. Reaching for a multicolored object while in an unsupported standing position B. Reaching for a multicolored object while in an unsupported, guarded sitting position . Visually tracking a black and white object held 9 inches from his/her face D. Reaching for a black and white object while in the supine position Correct Answer: B The appropriate task would include the 6-month-old gross developmental level activity of working on unsupported sitting. A multicolored object is appropriate for a 3-year-old cognitive level. Incorrect Choices: Standing and supine are not appropriate choices (too advanced or not advanced enough). The use of multicolored object is more appropriate than a black and while object for a 3-year-old cognitive level. 2. A patient is receiving mobilizations to regain normal mid thoracic extension. After three sessions, the patient complains of localized pain that persists for greater that 24 hours. What is the therapist's best option? ‘A. Change mobilizations to gentle, low-amplitude oscillati tissue irritation B. Continue with current mobilizations, followed by a cold pack to the thoracie spine C. Place the physical therapy on hold and resume in 1 week D. Change to self-stretching activities, because the patient does not tolerate mobilization Correct Answer: A Changing to low-amplitude oscillations will promote a decrease in the pain and tissue irritation. If pain persists for more than 24 hours, the soft tissue and joint irritation may progress. Incorrect Choices: Pain beyond 24 hours indicates possible tissue damage, so modification would be indicated. Placing the patient on hold would not be indicated or appropriate based on the patient's response. Self-stretching will improve the osteokinematic motion but not the arthrokinematic motion, so this would not be an appropriate modification. It is not specific to the joint and may increase the irritation. It certainly would not decrease the pain and irritation. to reduce the joint and soft 3. A therapist wishes to use behavior modification techniques as part of a plan of care to help shape the behavioural responses of a patient recovering from traumatic brain injury (TBI). What intervention is the BEST to use? A. Use frequent reinforcements for all desired behaviors B. Encourage the staff to tell the patient which behaviors are correct and which are not ~. Reprimand the patient every time an undesirable behavior occurs D, Allow the patient enough time for self-correction of the behavior Correct Answer: A Behavioral modification is best achieved through use of positive reinforcements for all desired behaviors. Incorrect Choices: Negative behaviors should be ignored, not reprimanded. Self-correction is not a form of behavior modification. 4. A patient with multiple sclerosis (MS) presents with dysmetria in both upper extremities. Which of the following interventions is the BEST choice to deal with this problem? A. 341b weight cuffs to wrists during activities of daily living (ADL) training B. Isokinetic training using low resistance and fast movement speeds C. Pool exercises using water temperatures greater than 85°F D. Proprioceptive neuromuscular facilitation (PNF) pattems using dynamic reversals with carefully graded resistance Correct Answer: D Dysmetria is a coordination problem in which the patient is unable to judge the distance or range of movement (overshoots or undershoots a target). Adding manual resistance with PNF can assist the patient in slowing down the movement and achieving better control. Incorrect Choices: The patient lacks speed control. Low-resistance, fast-speed isokinetic training is contraindicated. The resistance of water (pool therapy) could help control the speed of movements, but the temperature is too warm (patients with MS demonstrate heat intolerance). Weight cuffs could also help slow the movements down but would unnecessarily fatigue the patient (patients with MS demonstrate problems with excessive fatigue). 5. Knee capsular tightness has limited a patient’s ability to attain full flexion, An INITIAL intervention a physical therapist can employ to restore joint motion should emphasize sustained mobilization in the loose-packed position. Which of the following is the BEST choice to use? A. Anterior glide and external rotation of the tibia B. Posterior glide and external rotation of the tibia C. Posterior glide and internal rotation of the tibia D. Anterior glide and internal rotation of the tibia Correct Answer: C Posterior glide and internal rotation are accessory motions necessary to increase knee flexion. Initial treatment should not result in pain, soreness, or diminished range of motion. Incorreet Choices: Anterior glide and external rotation are motions necessary for knee extension 6. A patient recovering from cardiac transplantation for end-stage heart failure is referred for exercise training. The patient is receiving immunosuppressive drug therapy (cyclosporine and prednisone), What guidelines should the therapist follow when implementing an exercise program for this patient? A. Require longer periods of warm-up and cool-down B. Require short bouts of exercise C. Eliminate all resistance training D. Require a frequency of 2-3 times/week Correct Answer: A A patient recovering from cardiac transplantation will require longer periods of warm-up and cool-down because physiological responses to exercise and recovery take longer. Incorrect Choices: Low-to moderate-intensity resistance training can be performed. Aerobic exercise should be performed 4-6 times/week, while progressively increasing the duration of training from 15-60 minutes per session. 7. A patient presents with weakness and atrophy of the biceps brachii resulting from an open fracture of the humerus. The therapist reads a report of needle electromyography (EMG) of the biceps. What is the anticipated muscle response after the needle is inserted and prior to active contraction? A. Polyphasie potentials B, Interference patterns C. Electrical silence D. Fibrillation potentials Correct Answer: C Inserting an EMG needle into a normal muscle cause a burst of electrical activity (insertional activity) after which the muscle produces no sound (electrical silence). Incorrect Choices: Fibrillation potentials are spontaneous activity seen in relaxed denervated muscle, and polyphasie potentials are produced in the contracted muscle undergoing reorganization. 8. A new staff physical therapist (PT) on the oncology unit of a large medical center receives a referral for strengthening and ambulation for a woman with ovarian cancer. She is undergoing radiation therapy after surgical hysterectomy. Her current platelet count is 17,000. What intervention is indicated for this patient at this time? A. Active range-of-motion (AROM) exercises and activities of daily living (ADLs) exercises B. Aerobic exercise 3-5 days/week at 40-60%, one repetition maximum C. Resistance training at 60%, one repetition maximum D, Progressive stair climbing using a weighted waist belt Correct Answer: A AROM and ADL exercises are beneficial and safe for this patient. Incorrect Choices: Exercise testing and training is contraindicated in patients with cancer whose platelets are <50,000, WBC <3,000, Hemoglobin <10g/dL. Additional contraindications include significant bony metastases, severe cachexia, severe fatigue, or poor fianetional status. 9. A patient complains of increased pain and tingling in both hands after sitting at a desk for longer than I hour. The diagnosis is thoracic outlet syndrome (TOS). Which treatment would be the MOST effective physical therapy intervention? A. Cardiovascular training using cycle ergometry to reduce symptoms of TOS. B. Stretching program for the pectoralis minor and scalenes C. Strengthening program for the scalenes and stemocleidomastoids D. Desensitization by maintaining the should in abduction, extension, and external rotation with the head tumed toward the ipsilateral shoulder Correct Answer: B TOS is described as compression to the neurovascular structures in the scalene triangle, the area defined by the anterior and middle scalenes between the clavicle and the first rib. The compression is a result of a shortened pectoralis minor and scalene muscle. Therefore, a stretching program to these muscles to gain space in the scalene triangle is appropriate. Incorrect Choices: Shortening of the scalenes and sternocleidomastoids may be the culprit that caused TOS to develop. Strengthening theses muscles would not improve the amount of space in the scalene triangle space. Cardiovascular training, especially performed in the posture using a cycle ergometer, would not improve the disorder. The problem in TOS is too much vascular volume in too small space. Increasing the vascular volume through that space with cardiovascular exercise will not resolve the symptoms of TOS. Desensitization by putting the shoulder and neck in this position is likely to diminish the space in the scalene triangle and further compress the neurovascular structures that run through that triangle. 10.A patient with diabetes is exercising. The patient reports feeling weak, dizzy, and somewhat nauseous. The therapist notices that the patient is profusely and is unsteady when standing. What is the therapist's BEST immediate course of action? A. Insist that the patient sit down until the orthostatic hypotension resolves B, Have a nurse administer an insulin injection for developing hyperglycemia C. Administer orange juice for developing hypoglycaemia D. Call for emergency services; the patient is having an insulin reaction Correct Answer: C Hypoglycaemia, or abnormally low blood glucose, results from too much insulin (insulin reaction). It requires accurate assessment of symptoms and prompt intervention. Have the patient sit down and give an oral sugar (e.g., orange juice). Incorrect Choices: One the patient is stabilized, the physician should be notified. Emergency services are generally not needed. Profuse sweating does not usually accompany orthostatic hypotension. 11.A patient with postpolio syndrome started attending a supervised outpatient exercise program, The patient failed to show up for follow-up sessions. The patient reported increased muscle pain and being too weak to get out of bed for the past 2 days. The patient is afraid to contimue with the exercise class. What is the therapist’s BEST course of action regarding the patient's exercise program? A. Discharge the patient from the program because exercise is counterproductive in postpolio syndrome B. Reschedule exercise workouts for early moming when there is less fatigue C. Decrease the intensity and duration, but maintain a frequency of 3 time/week D. Decrease the frequency to once a week for an hour session, keeping the intensity moderate Correct Answer: C Clinical manifestations of postpolio syndrome include myalgias, new weakness as well as atrophy and excessive fatigue with minimal activity. Nonexhaustive exercise and general body conditioning are indicated. A change in the exercise prescription (intensity and duration) is warranted. Incorrect Choices: The patient should not exercise to the point of fatigue and exhaustion. A frequency of one a week is too little to be beneficial. Rescheduling exercise to early morning does not address the needed change in exercise prescription. Stopping exercise completely will not help this patient. 12.A therapist has elected to use continuous inductive coil short wave diathermy (SWD) as one of the interventions in managing hip pain. Use of other thermal or electrical modalities were either ineffective or contraindicated. Which patient would be a candidate for use of short wave diathermy? A. A morbidly obese patient B. A patient with Type I diabetes who uses an insulin pump C. An I] year-old boy with a slipped capital femoral epiphysis, D. A patient with ankylosing spondylitis on high doses of NSAIDS and DMARDS Correct Answer: D Ankylosing spondylitis is a form of arthritis that primarily affects the spine and other joints including the hip. Pain, stiffness, and inflammation can result. A physical therapy treatment plan can incorporate deep heating for muscle relaxation followed by stretching, posture management, and other exercises. Inductive SWD is preferred here as it penetrates more deeply than capacitive SWD. The use of the medications indicated will have no bearing on the application of SWD. Incorrect Choices: Collagen, fat, and bone have low conductivity. Synovial tissue and muscle have high conductivity and the most heat is produced in these tissues. Use of SWD with a morbidly obese patient with excessive amounts of fat in the hip region could result in selective overheating of adipose tissue because of poor conductivity of electromagnetic energy. The energy wouldn't effectively reach the joint and surrounding muscle. SWD is contraindicated for patients that have implanted devices such as pacemakers, neural stimulators, or insulin pumps as diathermy can interfere with the functioning of the device. It is also contraindicated with those patients that have open, growing epiphyses as diathermy may alter the rate of epiphyseal closure. 13.A patient is recovering from open heart surgery (sternotomy and coronary artery bypass). The PT is supervising the patient’s outpatient exercise program at 8 weeks postsurgery. What guideline should be followed regarding the use of moderate to heavy weights during resistance training? ‘A. Should include upper body exercises only B. Is contraindicated during the first two months C. Should be based on 60%-80%, one repetition maximum initially D. Can be included if resistance training is once a week Correct Answer: B Resistive training after cardiothoracic surgery is restricted to 5 to 8 pounds for the first 5 to 8 weeks. Moderate to heavy resistance exercises are contraindicated. Incorrect Choices: Resistance training can begin 5 weeks postsurgery, including 4 weeks of consistent participation in a supervised cardiac rehabilitation endurance training program. Once cleared, initial loads for the upper body should be 30%-40%, one repetition max, and 50%-60% for hips and legs. 14.A patient with asthma is taking a drug from the sympathomimetic group, albuterol (Ventolin). What is the MOST important effect of this medication? A. Increases airway resistance and decreases secretion production B, Reduces airway resistance by reducing bronchospasm C. Increases heart rate (HR) and BP to enhance training effect during aerobic activity D. Reduces bronchial constriction and high blood pressure (BP) that accompanies exercise Correct Answer: B Sympathomimetics are a class of drugs that mimics the effects of stimulation of body organs and structures by the sympathetic nervous system. Albuterol (Ventolin) has the primary action of reducing airway resistance by a decrease in bronchospasm. Incorrect Answer: Albuterol decreases airway resistance and has no effect on the volume or consistency of airway secretions. The primary effects of albuterol are on beta? receptors in the bronchiole smooth muscle. It may also have an effect on betal receptors, producing cardiovascular adverse reactions of increased BP and tachycardia. These adverse effect can result in a patient monitoring exercise parameters at lower exercise workloads and reducing aerobic training effects. 15.A PT decides to exercise a patient with lower extremity lymphedema using aquatic therapy. Hydrostatic pressure exerted by the water can be expected to do which of the following? A. Increase cardiovascular demands at rest and with exercise B. Reduce effusion and assist venous return C. Provide joint unloading and enhance ease of active movement D. Increase resistance as speed of movement increases Correct Answer: B The pressure exerted by water on an immersed object is equal on all surfaces (pascal’s law). As the depth of immersion increases, so does hydrostatic pressure. Increased pressure limits effusion, assists venous return, and can induce bradycardia, Incorrect Choices: The other choices do not relate directly to hydrostatic pressure. Buoyancy of water provides an environment of relative weightlessness and assists in joint unloading and active movement. Hydromechanics, movement of water molecules, increases the resistance of water as speed of movement increases. Hot water immersion (35°C) can increase cardiovascular demands at rest and with exercise. 16.A therapist determines that a patient is walking with a backward trunk lean with full weight on the right leg. The patient also demonstrates great difficulty going up ramps. What is the BEST intervention to remediate this problem? A. Strengthen hip extensors through bridging B. Stretch hip abductors through side-lying positioning C. Strengthen knee extensors with weights, using 80%, one repetition maximum D. Stretch hip flexors through prone-lying positioning Correct Answer: A Backward trunk lean (gluteus maximus gait) is the result of a weak gluteus maximus. It causes increased difficulty going up stairs or ramps. Functional-training exercises such as bridging are indicated. Incorrect Choices: The patient is able to perform a backward trunk lean in standing, indicating adequate range in hip flexors. Ability to take full weight on the limb without knee buckling indicates adequate strength of knee extensors. Tightness in hip abductors is rare and would result in the posture (lateral lean) being maintained during all phases of gait. 17.A patient has developed a thick eschar secondary to a full-thickness burn. What is the antibacterial agent MOST effective for infection control for this type of burn? A. Sulfamylon B. Nitrofurazone C. Panafil D. Silver nitrate Correct Answer: A Sulfamylon penetrates through eschar and provides antibacterial control. Incorrect Choices: Silver nitrate and nitrofurazone are superficial agents that attack surface organisms. Panafil is a keratolytic enzyme used for selective debridement. 18.An elderly person has lost significant functional vision over the past 4 years and complains of blurred vision and difficulty reading. The patient frequently mistakes images directly in front of her, especially in bright light. When walking across a room, the patient is able to locate items in the environment using peripheral vision when items are located to both sides. Based on these findings, what is the visual condition this patient is MOST likely experiencing? A. Glaucoma B. Cataracts C. Homonymous hemianopsia D. Bitemporal hemianopsia Correct Answer: B Cataracts, which cause a clouding of the lens, result in a gradual loss of vision; central vision is lost first, then peripheral. Incorrect Choices: Glaucoma produces the reverse symptoms: loss of peripheral vision occurs first, then central vision, progressing to total blindness. Hemianopsia is a field defect in both eyes that often occurs following stroke. There was no mention of cerebrovascular-accident (CVA) in the question. 19.4 patient presents with complaints of pain and difficulty with ADL that is consistent with carpal tunnel syndrome, What is the BEST test to identify the cause of symptoms in this patient? A. Pronator teres syndrome test B. Ulnar nerve tension test C. Alllen’s test D. Phalen’s test Correet Answer: D Phalen’s test places stress on the compartment where the median nerve passes into the hand, so this test is typically positive for patients with carpal tunnel syndrome. Incorrect Choic Allen’s test is utilized to identify appropriate blood flow of the radial and ulnar arteries into the hand. The pronator teres syndrome test also involves the median nerve, but patients with median nerve involvement caused by an entrapment oat the pronator teres will have signs and symptoms proximal to the hand as well. The ulnar nerve tension test (ULTTS) is utilized to determine an abnormal motion/glide for the ulnar nerve. None of these tests would duplicate the signs and symptoms of carpal tunnel syndrome. 20.A patient incurred a right CVA I month ago and demonstrates moderate spasticity in the left upper extremity (predominantly increased flexor tone). The major problem at this time is a lack of voluntary movement control, There is minimal active movement, with “4 inch subluxation of the shoulder. What initial treatment activity is the BEST choice for this patient? A. Sitting, left active shoulder protraction with extended elbow and shoulder flexed to 90° B. Sitting, weight bearing on extended left upper extremity, weight shifting ~. Quadruped, rocking from side to side D. PNF D2 flexion pattern, left upper extremity Correct Answer: B Sitting, weight bearing, and rocking on an extended left upper extremity will help to decrease the flexor tone. It also provides joint compression (approximation) at the shoulder, which will help maintain shoulder position and stimulate stabilizing muscles. Incorrect Choices: Quadruped is too strenuous for this patient at this time (maximum weight bearing on a weak, unstable upper extremity). The other two activities demand more voluntary control than this patient currently demonstrates. 21.An older patient complains of pain in the right hip region. The therapist suspects hip osteoarthritis based on the patient's subjective symptoms. What clinical test is the BEST choice to confirm this diagnosis’? A. Scouring test B. Thomas test C. Craig’s test D. Posterior impingement test Correct Answer: A ‘A positive scouring test would be a consistent finding for a patient who has osteoarthrosis of the hip joint. It compresses the joint. Incorrect Choices: Craig’s test is utilized to examine for increased or decreased anterior antetorsion angle. The Thomas test examines the flexibility of the hip flexor muscles. The posterior impingement test is utilized to examine for impingement of the posterior hip joint capsule and/or labrum. Therefore, none of these tests would produce positive findings consistent with hip osteoarthrosis. 22.The therapist in the photograph is testing which muscle? A. Upper trapezius B. Middle deltoid C. Supraspinatus D. Anterior deltoid Correct Answer: C The muscle being tested is the supraspinatus. The empty-can position puts the supraspinatus muscle in its most effective position for contraction. Weakness may be a result of inflammation, neuropathy of the suprascapular nerve, or a tendon tear. Incorrect Choices: The muscle test for the anterior deltoid would have the humerus and forearm in neutral rather than intemal rotation of the humerus and pronation of the forearm shown in the picture. The muscle test for the middle deltoid would have the shoulder in abduction to 90°. The muscle test for the upper trapezius is the shoulder shrug. 23.During an examination, a patient demonstrates large-amplitude, sudden flailing motions of the arm and leg on one side of the body with primary involvement of axial and proximal joint muscles. What clinical term BEST describes the patient's behaviors? ‘A. Chorea B. Intention tremor C. Hemiballismus D. Athetosis Correct Answer: C Hemiballismus refers to sudden, jerky, forceful, and flailing involuntary movements on one side of the body (“Hemi” was a clue). Incorrect Choices: Athetosis refers to slow, writhing, and twisting involuntary movements. Chorea refers to rapid, irregular, and jerky involuntary movements. Intention tremor refers to involuntary oscillatory movements that occur during voluntary movement, None of these are localized just to one side of the body. 24. patient is referred to physical therapy after an antero-inferior dislocation of the right shoulder. What positive examination finding is expected as a result of this dislocation? A. Weak rhomboids B. Positive drop arm test C. Positive Neer’s test D. Weak deltoids Correct Answer: D Because of the anatomical position of the axillary nerve, it ean be damaged by an antero- inferior dislocation at the glenohumeral joint. This results in weak deltoids. Incorrect Choices: A drop arm test evaluates the integrity of the rotator cuff, A Neer’s test evaluates impingement of the shoulder. The rhomboids are innervated by the dorsal scapular nerve. Anatomically, the dorsal scapular nerve is medial and posterior to the shoulder joint. 25.A team of researchers investigates the use of constraint-induced movement therapy on patients with chronic stroke (1 year poststroke) using a multicentre randomized controlled trial (RCT), What are the specific characteristics of this type of research design’? A. A sample of convenience for the intervention group B. Alternating experimental and control conditions for a subject C. Random assignment to an experimental or control group D. Random assignment to matched cohort groups Correct Answer: C An RCT uses a randomization process to assign subjects to either an experimental group(s) or a control (comparison) group. Subjects in the experimental group receive the intervention and are then compared with subjects in the control group who do not receive the intervention. A large multicentre RCT study with large numbers of patients provides the highest level of scientific rigor and evidence. Incorrect Choices: ‘Alternating experimental and control conditions for a subject is an A-B-A-B design typically used in single-subject design studies. A cohort design investigates a group of subjects without a control group (sample of convenience). 26.A therapist has been asked to give an in-service presentation to staff aides on safe guarding techniques in a mursing home. The patients are at risk for falls. How should the therapist BEST prepare for this talk? A. Provide a questionnaire to a random sampling of participants 1 week before the scheduled presentation B. Provide a questionnaire to all participants 2 weeks before the scheduled session C. Survey the audience a day before the scheduled session D. Survey the audience at the scheduled session Correct Answer: B A questionnaire to all participants represents the best method of needs assessment in this situation. Incorrect Answer: Leaving the survey to the day of or day before the presentation does not allow for adequate advanced planning to meet the group’s needs. Using a random sample of participants will not adequately represent the needs of the whole group. 27.A therapist is treating a patient with left hemiplegia and profound visuospatial perceptual deficits. What is the BEST strategy to use initially to assist this patient in the relearning of motor tasks? A. Simplify and restructure the environment and minimize distractions B. Maximize use of demonstration and gesture C. Minimize use of verbal cues D. Encourage independent practice Correct Answer: A Working in a closed environment in which clutter and distractions are minimized or eliminated is the best choice for this patient during initial training. Incorrect Choices: Patients with visuospatial deficits respond best to consistent, simplified verbal cues. Demonstration and gesture are likely to be confusing. Supervised practice is best initially because patients tend to be impulsive and unrealistic about their residual abilities. 28.A 2-week-old infant born at 27 weeks gestation with hyaline membrane disease is referred for a physical therapy consult. Nursing reports that the child “desaturates to 84% with handling” and has minimal secretions at present. What is the therapist’s BEST course of action? A. Provide suggestions to nursing for positioning for optimal motor development B. Put the PT consult on hold because the child is too ill to tolerate exercise C. Delegate to a physical therapy assistant (PTA) a maintenance program of manual techniques for secretion clearance. D. Perform manual techniques for secretion clearance, 2-4 hours daily, to maintain airway patency. Correct Answer: A Excessive handling of a premature infant can cause oxygen desaturation. It is in the best interests of the infant to limit the number of handlers. The PT’s role should be to assist nursing in developing positioning schedules, positions for feeding, infant stimulation activities, etc. Incorrect Choices: At present, there is little information provided that would necessitate the PT or PTA to be a direct caregiver to this child. 29.A patient sustained a valgus stress to the left knee while skiing. The orthopaedist found a positive McMurray’s test and a positive Lachman’s stress test. The patient has been referred to physical therapy for conservative management of this problem. What is the BEST intervention for the subacute phase of rehabilitation? ‘A. Open-chain exercises of the hip extensors and hamstrings to inhibit anterior translation of the femur on the tibi B. Closed-chain functional strengthening of the quadriceps femoris and hamstrings, emphasizing regaining terminal knee extension. C. Closed-chain functional strengthening of the quadriceps femoris and hip abductors to promote regaining terminal knee extension, D. Open-chain strengthening of the quadriceps femoris and hip adductors to inhibit anterior translation of the tibia on the femur. Correct Answer: B The evaluation is suggestive of an unhappy triad injury. Closed-chain exercises are emphasized during the subacute phase to enhance functional control of the muscles surrounding the knee. Terminal extension must be achieved during this stage if normal function is to occur. Incorrect Choices: Open-chain exercise does not promote regaining fimnetional control for the muscle surrounding the knee. Focus on the hip abductors (rather than the hamstrings) will not promote regaining functional control of the knee joint. 30.A patient with pain in the left lateral face and head is found to have limited active and passive mouth opening range of motion. However, passive lateral deviation is full to both sides. What is the likely reason for the limitation in mouth opening range of motion? A. Ananteriorly displaced disc with reduction in the left temporomandibular joint B, Decreased flexibility in the muscles of mastication on the left C. Capsular restriction of the left temporomandibular joint D. An anteriorly displaced disc without reduction in the left temporomandibular joint Correct Answer: B ‘Mouth opening requires lengthening of the muscles of mastication as the body of the mandible moves away from the upper palate. Lateral deviation does not require a significant lengthening in the muscles of mastication, as the primary motion of the mandible is a slight anterior translation of the mandibular condyle without increasing the distance between the body of the mandible and the upper palate. Incorrect Choices: Unilateral capsular and interarticular restrictions of the temporomandibular joint would result in a deflection of the mandible toward the side of restriction with opening and would limit lateral deviation away from the side of restriction due to a decreased anterior translation of the mandibular condyle. An anterior dise displacement with reduction would not limit mouth opening. 31.A patient with TBI has a convulsive seizure during a therapy session. The patient has lost consciousness and presents with tonic-clonic convulsions of all extremities. What is the therapist's BEST response? A. Position in supine-lying with head supported with a pillow, and wait out the seizure B. Wrap the limbs with a sheet to prevent self-harm, position in supine-lying and call for emergency assistance. C. Position in side-lying, check for an open airway, and immediately call for emergency assistance D. Initiate rescue breathing immediately and call for help to restrain the patient Correct Answer: C This is an emergency situation. In order to prevent aspiration, turn the head to the side or position in side-lying. Check to see whether the airway is open, and then call for emergency assistance. Wait for tonic-clonic activity to subside before initiating artificial ventilation, if needed. Incorrect Choices: Supine positioning can be life-threatening if the tongue falls backward to restrict the airway. Rescue breathing and restraining the patient are not indicated. 32.A therapist wants to know whether neurodevelopmental treatment (NDT) handling techniques produce an improvement in independent rolling that lasts longer than 30 minutes. In this study, rolling is what type of variable? A. Control variable B. Independent variable C. Dependent variable D. Intervening variable Correct Answer: C ‘The dependent variable is the change or difference in behavior (in this example, rolling) that results from the intervention. Incorrect Choices: NDT handling is the independent variable. The terms “intervening” and “control” are not used to correctly define the study. 33.An adult with no significant past medical history presents to the emergency room with complaints of fever, shaking chills, and a worsening productive cough. The patient has chest pains over the posterior base of the left thorax, which are made worse on inspiration. What would be an expected physical finding for this patient? A. Symmetrical breathing B. Crackles over the left thorax C. Increased chest excursion D. Slowed respiratory rate Correct Answer: B Crackles are a typical finding over the area correlating with an infiltrate. Incorrect Choices: With a lower than normal tidal volume, a respiratory rate would have to be elevated, not slowed, to maintain an adequate minute ventilation (respiratory rate times tidal volume = minute ventilation). Because the patient is having pain, thoracic expansion would likely be limited and asymmetrical. 34.A patient is referred to physical therapy for functional gait difficulties. The patient is unable to take a normal step and drags the left foot. Examination reveals muscle weakness with fasciculations in the left lower leg. For what should the therapist examine? A. Decreased tone and hyporeflexia B. Muscle spasms and positive Babinski . Increased tone and hyperreflexia D, Dyssynergia and timing deficits Correct Answer: A Muscle weakness with fasciculations is symptomatic of a lower motor neuron (LMN) lesion. Other signs and symptoms of a LMN lesion include hypotonia or flaccidity, hyporeflexia, or absent reflexes and neurogenic atrophy. Incorrect Choices: The other choices are all signs and symptoms of upper motor neuron (UMN) lesions. 35.An elderly patient with hyperthyroidism is referred to physical therapy following a period of prolonged bedrest. What should the therapist be alert for when monitoring exercise of this patient? A, Decreased heart rate and blood pressure B. Tachycardia and dyspnea CC. Muscle weakness and joint pain D. Ammhythmias and bradycardia Correct Answer: B Hyperthyroidism is a hypermetabolic state and is associated with exercise intolerance and impaired cardiopulmonary function. Symptoms include dyspnea, fatigue, tachycardia, and arrhythmia, In older people there is increased risk of aggravating pre-existing heart disease (eg., atrial fibrillation, angina, and myocardial infarction). Incorrect Choices: Heart rate is increased during exercise in hyperthyroidism. Muscle weakness (typically proximal) and fatigue are present while increased joint pain is not characteristic. Tachycardia, not bradycardia, is seen. 36.A patient strained the lower back muscles 3 weeks ago, and now complains of pain (6/10). Upon examination, the therapist identifies bilateral muscle spasm from T10-L4. The therapist elects to apply interferential current to help reduce pain and spasm, What is the BEST electrode configuration in this case? A. Four electrodes, with current flow perpendicular to the spinal column B. Two electrodes, with current flow perpendicular to the spinal column C. Four electrodes, with current flow diagonal to the spinal column D. Two electrodes, with current flow parallel to the spinal column Correct Answer: C The crisscrossed electrode configuration allows: (1) a greater area to be treated and (2) current interference to occur between the frequencies of the two circuits because of the diagonal pattern. Incorreet Choices: ‘A crisscrossed electrode configuration is needed to create interferential current. None of the other electrode configurations facilitates the flow of current diagonal to the spinal column, which would cause the frequencies to intersect. 37.An infant is independent in sitting, including all protective extension reactions, and can pull- to-stand through kneeling, cruise sideways, and stand alone, The infant still demonstrates plantar grasp in standing. What is this infant's approximate chronological age? ‘A. 6 months B. 5 months C. 8-9 months D. 10-15 months Correct Answer: C The 8 to 9-month-old will be able to pull-to-stand, stand alone, and cruise sideways, but because he/she is not yet walking, may still exhibit plantar grasp in the standing position. Incorrect Choices: ‘At S months, the infant can roll prone to supine and demonstrates head control in supported sitting. At 6 months, the infant can sit independently and pull-to-stand. At 10-15 months, the infant typically begins to walk unassisted. 38.A retired bus driver has experienced increasing frequency of low back pain over the past 10 years. The patient states that nonsteroidal anti-inflammatory drugs (NSAIDs) help to relive the symptoms, but there is always a nagging-type pain, the patient reports significant stiffness in the morning that dissipates by noon after exercising and walking. Pain is exacerbated with frequent lifting and bending activities, as well as sitting for long period. What should the physical therapy plan of care emphasize? A. Modalities to reduce pain, postural re-education, and dynamic stabilization exercises. B. Postural re-education, soft tissue mobilization, and dynamic stabilization. C. Modalities to reduce pain, joint mobilization, and lumbar extension exercises. D, Joint mobilization, soft tissue mobilization, and flexion exercises. Correct Answer: B This is a long-term degenerative and postural dysfunction that is manageable with medication and proper physical activity. Therefore, the most effective use of treatment time should emphasize regaining normal postural alignment and functional ADLs. Incorrect Choices: ‘Whereas modalities and mobilization can relieve acute pain, this is a chronic problem that demands choices appropriate for long-term management of the problem. 39.A patient presents to physical therapy with a primary complaint of low back pain and right lower extremity radicular symptoms extending distally to the calf of 2 weeks duration. Current pain intensity is rated as 2/10 with rest and 5/10 during lumbar extension movements. What is the strongest prognostic indicator that would affect the clinical outcome? A. 2/10 pain intensity with rest B. Current symptom duration (2 weeks) C. Lower extremity radicular symptoms D. 5/10 pain intensity during lumbar extension Correct Answer: C The presence of lower extremity radicular symptoms in patients experiencing low back pain is a strong negative prognostic indicator for achieving good clinical outcomes. Incorrect Choices: Pain intensity with rest in this case was fairly low (ie., 2/10) and this pain intensity assessment alone may be limited because it does not consider pain intensity during activity. An increase in pain intensity with lumbar extension movement (ie., 5/10) does provide important information relevant to activity; however, it does not necessarily indicate a poor prognosis alone and is not as strong a negative prognostic indicator when compared to lower extremity radicular symptoms. Symptom duration of 2 weeks is not considered a chronic state; therefore, it may not be viewed as a negative prognostic indicator for clinical outcomes. 40.An elderly patient with diabetes and bilateral lower extremity amputation is to be discharged from an acute care hospital 2 weeks postsurgery. The incisions on the residual limbs are not healed and continue to drain. The patient is unable to transfer because the venous graft sites in the upper extremities are painful and not fully healed. Endurance out-of-bed is limited. What is the BEST choice of discharge destination for this patient? A. Skilled nursing facility B. Custodial care facility C. Home D. Rehabilitation hospital Correct Answer: A A skilled nursing facility is the best facility because the patient continues to require nursing care for the open wounds. Initiation of physical therapy when this patient is able is also available Incorrect Choices: Discharge to home would be premature because the patient is unable to transfer. Custodial care involves medical or nonmedical care that does not seek a cure. A rehabilitation hospital is not appropriate at this time, because the patient cannot actively participate in rehabilitation 3 hours/day. 41.A patient currently being seen for low back pain awoke one moming with drooping left facial muscles and excessive drooling. The patient was recovering from a cold and had experienced an earache in the left ear during the previous 2 days. The therapist suspects Bell’s palsy. What cranial nerve test can confirm this diagnosis’? A. Taste over the posterior tongue, and having the patient protrude the tongue B. Taste over the anterior tongue, and having the patient raise the eyebrows and puff the cheeks C. Corneal reflex and stretch reflexes of facial muscles D. Trigger points for pain, especially over the temporomandibular joint (TMJ). Correct Answer: B Bell’s palsy is a lower motor neuron lesion affecting the branches off the facial nerve, CN VII. Examination of the motor function of the muscles of facial expression (i.e., raise eyebrows, show teeth, smile, close eyes tightly, puff cheeks) and taste over the anterior tongue will reveal deficits of CN VII function. Incorrect Choices: Taste over the posterior tongue is a function of CN IX (glossopharyngeal). Strength of tongue protrusion is a function of CN XII (hypoglossal). Pupillary reflexes are a function of CN II (optic). 42.A PTA is assigned to ambulate a patient with a 10-year history of Parkinson’s disease (PD). What should the PT instruct the PTA to watch for? A. Wider steps and increased double support time B. An abnormally wide base of support C. Decreased trunk rotation with shorter steps D. Unsteady, uneven gait with veering to one side Correct Answer: C Gait changes characteristic of PD include loss of arm swing and reciprocal trunk movement, shuffling gait with shorter steps, and festinating gait (an abnormal and involuntary increase in the speed of walking), Incorrect Choices: Patients with PD take shorter steps (not longer). Veering to one side only is indicative of a unilateral peripheral vestibular deficiency (the patient veers to the side with the dysfunction). ‘An abnormally wide base of support is indicative of gait unsteadiness but is not typical in the patient with PD. 43.A patient with congestive heart failure (CHF) is on a regimen of diuretics (chlorotiazide).. What are the potential adverse effects of this medication that the PT should be alert for? A. Hyperkalemia and premature ventricular contractions (PVCs). B. Myalgia and joint pains C. Orthostatic hypotension and dizziness D. Reflex tachycardia and unstable BP. Correct Answer: C Thiazide diuretics are used to manage mild to moderate hypertension. Adverse side effects include orthostatic hypotension and dizziness, along with drowsiness, lethargy and weakness. These represent a safety risk during functional training and gait. Incorrect Choices: BP is lowered and is more stable, not less. Hypokalemia (not hyperkalemia) can occur, resulting in increased PVCs, Muscle cramps and weakness can occur. Joint pains are likely caused by a comorbid condition. 44. patient with a traumatic injury to the right hand had a flexor tendon repair to the fingers. When should physical therapy intervention being following this type of repair? A. After the splint is removed in 4-6 weeks to allow ample healing time for the repaired tendon B. After the splint is removed in 2-3 weeks to allow full AROM of all affected joints C. Within a few days after surgery to allow for early initiation of strengthening exercises D. Within a few days after surgery to preserve tendon gliding Correct Answer: D Early passive and active assistive exercises promote collagen remodelling to allow free tendon gliding Incorrect Choices: When rehabilitation is delayed by several weeks, adhesions form, which restrict free tendon gliding. Early initiation of strengthening exercises is contraindicated. 45.A female patient complains of intermittent pain in the right sacroiliac region. There was an insidious onset approximately 4 months ago. Pain has gradually worsened and is now fairly constant and does not vary much with activity or movement. Active motion assessment of the lumbar spine reveals no change in symptoms with movement. Sacroiliac provocation tests are negative, The patient is mildly tender over the right sacroiliac region. What is the MOST likely diagnosis for this patient? A. Sacroiliac joint sprain B. Multifidus muscle strain C. Ovarian cyst D. Right L5/S! facet joint arthrosis, Correct Answer: C ‘An ovarian cyst can refer pain to the sacroiliac region and is more likely to cause constant pain that dies not vary much with activity. Symptoms are not likely to be reproduced with a musculoskeletal examination, Incorrect Choices: A sacroiliac joint strain, LS/SI facet arthrosis, and multifidus musele strain should cause reproduction of the patient’s symptoms with active movements. The symptoms should be activity related. They will therefore vary in intensity depending on the particular activity in which the patient engages. In addition, a sacroiliac sprain would likely be symptomatic with SI provocation tests. 46.The therapist suspects that a patient recovering from a middle cerebral artery stroke is, exhibiting a pure hemianopsia, What test should be used to confirm the hemianopsia? A. Penlight held approximately 12 inches from the eyes and moved to the extremes of gaze right and left B. Penlight held 6 inches from the eyes and moved inward toward the face C. Visual confrontation test with a moving finger D. Distance acuity chart placed on a well-lighted wall at patient’s eye level 20 feet away Correct Answer: C Visual field is examined using the confrontation test. The patient sits opposite the therapist and is instructed to maintain his/her gaze on the therapist’s nose. The therapist slowly brings a target (moving finger or pen) in the patient’s field of view alternately from the right or left sides. The patient indicates when and where he/she first sees the target. Incorrect Choices: Distance acuity vision is tested using a Snellen eye chart at a distance of 20 feet. Ocular pursuit is tested using a penlight moved in an H pattem to the extremes of gaze. Convergence is tested using a penlight and ruler; the patient keeps the penlight in focus as it moves inward froma distance of 4 or 6 inches. 47.An overweight adult patient complains of right anterior hip and knee pain while walking, especially when weight bearing on the right. Lumbar AROM is normal and pain free. Right hip AROM and PROM are limited compared to the left. Right knee AROM and PROM are full and pain free. There is no pain with resisted testing at the right hip or right knee. The scouring test reproduces the patient’s hip and knee symptoms. Hip joint distraction relieves these symptoms. Based on the above findings, what is the MOST likely diagnosis? A. Trochanteric bursitis B. Patellofemoral syndrome C. Piriformis strain D. Hip degenerative joint disease. Correct Answer: D Scouring test or compression at the hip joint reproduces the patient’s symptoms, suggesting a joint problem at the hip. The scouring test can also result in referred pain to the knee. Weight bearing that compressed the hip joint also reproduces the patient’s symptoms. Distraction of the hip joint surfaces relieves symptoms, suggesting hip DID. The anterior knee region is a common pain referral site for the hip joint. In addition, the overweight condition supports the diagnosis of hip DID due to excessive compressive forces over an extended period of time. Incorrect Choices: There is no pain with resisted testing, which rules out a contractile problem. Knee AROM, PRM, and resisted testing are negative, ruling out patellofemoral syndrome. Pain from trochanteric bursitis is specifically localized to the greater trochanter. 48.A therapist investigated the accuracy of pulse oximetry estimates during exercise. Correlational analysis measured the strength of the relationship between two types of ear probe-equipped pulse oximeters during heavy cycle exercise under hypoxic conditions. The investigator found measured arterial oxyhemoglobin saturation (%HbO:) level to have a correlation of 0.89 at high saturation but only 0.68 at low saturation levels. How should the therapist interpret these results? A. During heavy exercise, oxygen saturation levels should be interpreted cautiously B. Both devices are highly accurate at all saturation levels C. Accuracy of the measurements increases at higher saturation levels D. Both devices are only moderately accurate Correct Answer: C The result of the study indicates that the correlation between the two types of oximiters was high when oxygen saturation levels were high (0.89), but only moderate (0.68) at low oxygen saturation levels. Incorrect Choices: Accuracy was not the same at all saturation level, and the high correlation during high saturation suggests that the devices are accurate during heavy exercise. 49.A college soccer player sustained a hyperextension knee injury when kicking the ball. The patient was taken to the emergency room of a local hospital and was diagnosed with “knee sprain.” The player was sent to physical therapy the next day for rehabilitation, As part of the examination to determine the type of treatment plan to implement, the therapist conducted the test shown in the figure. Based on the test picture, the therapist is examining the integrity of which structure? A. Iliotibial band B. Posterior cruciate ligament C. Anterior cruciate ligament D. Medial meniscus Correct Answer: C The test shown in the figure is Lachman’s stress test to determine the integrity of the anterior cruciate ligament Incorrect Choices: The posterior cruciate is examined using the posterior drawer and the reverse Lachman’s stress test. The medial meniscus is examined using MeMurray’s and Apley tests. The iliotibial band is tested using the Noble compression test. 50.A patient is seen in physical therapy 2 days after a motor vehicle accident. The chief complaints are headaches, dizziness, neck pain with guarding, and a “sensation of a lump in the throat.” Plain film x-rays were read as negative. The therapist should refer this patient for what type of imaging? A. Second series of plain film x-rays B, T2 magnetic resonance imaging (MRI) C. Computed tomography (CT) scan D. Myelogram Correct Answer: C The primary concern of the therapist is to rule out strong suspicions of an upper cervical spine fracture, CT scan is still preferred for assessing cortical bone, especially spinal fractures. Incorrect Choices: Plain films, already taken, did not show any fracture, which is not uncommon. A second series would not be expected to reveal any new information. The T2 MRI and myelogram are not as, specific for assessing bony anatomy as the CT sean. 51.During a test of upper extremity rapid alternating movement (RAM), the movements of the hands and elbows become irregular with wider excursions than expected. As speed is increased, the movements become more disorganized. These findings are indicative of: A. Brainstem dysfunction B. Lower motor neuron weakness C. Cerebellar dysfunction D. Upper motor neuron weakness Correct Answer: C Cerebellar dysfunction is characterized by classic cerebellar movement disturbances of dyssynergia (in this case), dysmetria and dysdiadochokinesia. Movement decomposition is velocity dependent, with greater disturbances in movement control at higher speeds. Incorrect Choices: UMN lesions can produce weakness and dyssynergia accompanied by spasticity. Spasticity is velocity dependent, with slowing of movements at faster speeds. LMN lesions produce weakness or paralysis, with hypotonia or flaccidity. Brainstem dysfunction produces a variety of deficits, with mixed sensory/motor symptoms. alll of these findings are not reported in this, case. 52. patient presents with a persistently downwardly rotated and adducted scapula during humeral elevation. The plan of care includes stretching and strengthening to improve range of motion. What muscles should be stretched and strengthened? A. Stretching rhomboid muscles and strengthening serratus anterior muscle B. Stretching pectoralis major and strengthening rhomboid muscles C. Stretching pectoralis minor and strengthening trapezius muscles D, Stretching serratus anterior and strengthening levator scapula and lower trapezius Correct Answer: A The rhomboids are scapular downward rotators, scapular adductors, and scapular elevators. Insufficient length of the rhomboids would limit upward scapular rotation, and stretching to restore proper length of these muscles would help to promote upward scapular rotation. Incorrect Choices: The serratus anterior muscles acts as an upward scapular rotator and scapular abductor. Weakness of the serratus anterior would impair active scapular upward rotation, and strengthening exercises would promote upward scapular rotation. 53.A patient is recovering from stroke and, at 4 months, is ambulating with a straight cane for household distances. During outpatient physical therapy, the therapist has the patient practice walking with no assistive device. Recurvatum is observed that worsens with continued walking. What is the therapist's BEST choice for intervention? A. Give the patient a small-based quad cane (SBQC) to improve stability and have hinvher practice AROM in supine B. Exercise the quadriceps using isokinetic resistance at higher loads and increasing speeds C. Practice isolated small-range quadriceps eccentric control work in standing and continue with the straight cane D. Give the patient a KAFO to control the hyperxtension and a hemi-walker. Correct Answer: C Eccentric quadriceps control work (closed-chain exercises) is indicated to reduce recurvatum. The patient should continue with a straight cane until able to walk without the device and recurvatum, Incorrect Choices: Open-chain exercises (isokinetic resistance, AROM) do not adequately address the functional demands of gait, The use of SBQC or hemi walker will not correct the problem. A KAFO is inappropriate to stabilize the knee which can be effectively stabilized using either an AFO or a Swedish knee cage. The use of an orthosis should be considered only as a last resort. 54.A patient recovering from an incomplete spinal cord injury at the L3 level (ASIA scale D) ambulates with bilateral Lofstrand crutches. The patient reports great difficulty going down ramps with unsteady; wobbly knees. What is the BEST intervention to use with this patient? A. Prolonged icing to reduce hamstring pain B. Stretching using a posterior resting splint for tight plantar flexors C. Progressive resistance training for the quadriceps D. Biofeedback training to reduce knee extensor spasticity Correct Answer: C Aspinal cord injury at the level of L3 affects knee extensors. ASIA scale D means the injury is incomplete, with at least half of the key muscles below the neurological level having a muscle grade of 3 or more. A weak knee will wobble or buckle going down stairs or ramps. It is the result fo weak quadriceps or knee flexor contracture. Strengthening exercises using progressive resistance training for the quadriceps are indicated. Incorrect Choices: Biofeedback training may reduce knee extensor spasticity, but this may only increase knee instability and is not indicated in this case. There is no indication that hamstring pain or tight plantarflexors are present or precipitating causes of the patient's problem. 55.Under HIPAA rules, to whom is it illegal to release protected health information (PHI) without a competent patient’s consent? A. A State Agency responsible for investigating suspected abuse B. The insurance company that is paying for the patient's treatment C. Another health care provider involved in the care of the patient D. The patient's spouse Correct Answer: D A spouse does not have the legal right to the patient's information without the patient’s consent Incorrect Choices: Those individuals involved in the care of the patient, a legal guardian with power of attorney in situations in which the patient is judged mentally incompetent, or the patient’s payer have a legal right to information regarding a patient's care without obtaining the patient’s consent for releasing information. The therapist has a positive legal obligation to report suspected abuse whether or not consent is granted. 56.With respect to a worker's sitting posture, the greatest reduction in lumbar spine compression forces would be achieved by: A. Eliminating armrests on the chair B. Decreasing the chair backrest-seat angle to 85° C. Increasing the chair backrest-seat angle to between 90° and 110° D. Using a 2-inch gel seat cushion Correct Answer: C ‘Maximal reduction of lumbar disc pressure can be achieved by increasing the angle between the seat pan and the chair backrest to between 90° and 110°, using armrests for support, or adding a lumbar support. Combining the effects of all three provides the best solution. Incorrect Choices: A gel seat cushion reduces pressures on the ischial seat. Eliminating armrests or decreasing the chair backrest-seat angle only worsen the problem. 57.A baseball pitcher is referred to physical therapy with progressive posterior shoulder pain and weakness of the shoulder abductors and lateral rotators. The therapist notices muscle wasting superior and inferior to the scapular spine. The patient’s problem is MOST LIKELY attributable to damage involving the: A. Spinal accessory nerve B. Scalene muscles C. Suprascapular nerve D. Long head of the biceps brachii Correct Answer: C Microtrauma to the suprascapular nerve can occur with repetitive activities involving shoulder “cocking” and follow-through resulting in inflammation and muscle weakness of the muscles supplied by the suprascapular nerve (the supraspinatus and infraspinatus). Incorrect Choices: Damage to the spinal accessory nerve will promote weakness and atrophy of the upper trapezius muscle. Damage to the long head of the biceps brachii or scalene muscles will not present with posterior shoulder pain, weakness with shoulder abduction/external rotation, and/or atrophy of the supraspinatus and infraspinatus muscles. 58.An elderly patient presents with severe COPD, GOLD stage 4, shows which of the following physical examination findings would the therapist expect to find? A. Kyphosis with an increased thoracic excursion B. Barreled chest with a decreased thoracic excursion " Pectus excavatum with an increased thoracic excursion, D, Pectus carinatum with decreased thoracic excursion Correct Answer: B A patient with severe COPD (GOLD 4) will have lost much of the elastic recoil properties of the lung. The usual elastic properties of the lung tissue help to pull the thorax into the normal chest wall configuration of health. Without these elastic recoil properties the patient's thorax will “barrel” in appearance, meaning it is langer and rounder than what you would normally expect. As the thorax has moved into an inspiratory position at rest, there is less movement available, so a decreased thoracic excursion would be expected. Incorrect Choices: Pectus excavatum (funnel chest) is not an acquired chest wall deformity that results in decreased thoracic excursion. Pectus carinatum (pigeon breast) is not an acquired chest wall deformity that results in a decreased thoracic excursion. While the barrelling of the chest of COPD often has a kyphosis associated with it, the second hallmark to the chest wall deformity of COPD is a decrease in excursion. 59.A patient is recovering from surgical resection of an acoustic neuroma and presents with symptoms of dizziness, vertigo, horizontal nystagmus, and postural instability. To address these problems, what should the physical therapy plan of care incorporate? A. Repetition of movements and positions that provoke dizziness and vertigo B. Hallpike’s exercises to improve speed in movement transitions C. Strengthening exercises focusing on spinal extensors D, Prolonged bedrest to allow vestibular recovery to occur Correct Answer: A In patients with unilateral vestibular pathology, habituation training (use of positions and movements that evoke symptoms) will encourage the vestibular system to recalibrate. Good recovery can generally be expected with gradual progression of exercises. Incorrect Choices: Prolonged bedrest will delay recovery and may result in incomplete recovery. Strengthening exercises for spinal extensors are not indicated in the case information, Hallpike-Dix maneuver is used for assessment and diagnosis of benign paroxysmal positional vertigo and is not a set of exercises. 60.A physical therapist is treating a patient with diabetic peripheral neuropathy. The patient recently began taking Lyrica (pregabalin). During a monofilament exam of the feet the therapist notices circumferential marks bilaterally at the level of the malleoli after the socks are removed. The patient is complaining of increased difficulty ambulating long distances. In this situation, what is the therapist’s BEST course of action? A. Contact the physician about possible development of congestive heart failure B. Begin manual lymphatic drainage for secondary lymphedema ~. Complete the examination and instruct in proper skin care precautions D, Educate the patient about the risks of foot ulceration Correct Answer: A It is important to recognize serious side effects of commonly used medications and to institute contact with the physician as appropriate. Lyrica is used to help treat diabetic neuropathy. Serious side effects include heart failure, greater difficulty walking long-distances, and lymphedema (marks from the sock). Incorrect Choices: Although patient education and manual lymphatic drainage can be important parts of treatment, they are not as important as recognizing the potential for heart failure. 61.A patient presents to the clinic with pain and decreased fumetion of the right shoulder. A full tear of the right rotator cuff musculature is suspected. The special test that would provide the most valid and reliable information confirming this suspicion would be the: A. Drop arm test B. Yergason’s test C. Neer’s impingement test D. Clunk test Correct Answer: A The primary intent of the drop arm test is to assess if the rotator cuff is intact. If the rotator cuff is fully torn, the drop arm test will be positive, meaning that the arm will fall from a fully elevated position if it is unsupported. The Empty-can test is also a test for the rotator cuff muscles but is more specific for the supraspinatus. Incorrect Choices: ‘Yergason’s test assess if the transverse ligament is intact. Neer’s impingement test assesses for impingement of the rotator cuff, so once the tendon is torn this will no longer be positive. The chunk test is described as a test to assess for a glenoid labrum tear. None of these tests would be positive with a full tear of the rotator cuff, 62.A patient who is 5 weeks” post myocardial infarction (MI) is participating in a cardiac rehabilitation program. The therapist is monitoring responses to inereasing exercise intensity. The indicator that exercise should be immediately terminated is A. 1.5 mm of downsloping ST segment depression B. Peak exercise HR > 140 C. Appearance of a PVC on the electrocardiogram (ECG) D. Systolic BP > 140 mm Hg or diastolic BP > 80 mm Hg Correct Answer: A The upper limit for exercise intensity prescribed for patients post-MI is based on signs and symptoms. Of the choices, only ST segment depression (>1.0 mm of horizontal or downsloping depression) is a significant finding, representative of myocardial ischemia. Incorrect Choices: Both HR and BP are expected to rise (the levels of 140 and 140/80 are not significant for most patients). The appearance of a single PVC is also not significant because single PVCs can occur in individuals without a cardiac history. 63.An infant who was 39 weeks gestational age at birth and is now 3 weeks chronological age demonstrates colic. In this case, what is the BEST intervention the PT should teach the mother? A. Stroking and tapping B. Neutral warmth . Visual stimulation with a colored object D. Fast vestibular stimulation Correct Answer: B Neutral warmth achieved through wrapping or bundling the infant is a calming stimulus. Incorrect Choices: All of the other choices would likely increase arousal of the infant. The infant is still too developementally immature for any of the stimuli other than neutral warmth. 64.A therapist is treating a patient with Brown-Sequard syndrome that resulted froma gunshot wound. The therapist’s examination should reveal: A. Sparing of tracts to sacral segments with preservation of perianal sensation and active toe flexion B. Loss of motor function and pain and temperature sensation, with preservation of light touch and proprioception below the level of the lesion C. Loss of motor function below the level of the lesion, primarily in the upper extremities D. Ipsilateral loss of motor function, ipsilateral loss of light touch and proprioception, and contralateral loss of pain and temperature Correct Answer: D Brown-Sequard syndrome is a hemisection o f the spinal cord characterized by ipsilateral loss of dorsal columns with loss of touch, pressure, vibration and proprioception; ipsilateral loss of corticospinal tracts with loss of motor function below level of lesion; contralateral loss of spinothalamic tract with loss of pain and temperature below level of lesion; at lesion level bilateral loss of pain and temperature. Incorrect Choices: Anterior cord syndrome: loss of lateral corticospinal tracts with bilateral loss of motor function; loss of spinothalamic tracts with bilateral loss of pain and temperature; preservation of dorsal columns (proprioception, vibratory sense). Central cord syndrome: loss of spinothalamic tracts with bilateral loss of pain and temperature; loss of ventral horn with bilateral loss of motor function (primarily the upper extremities); preservation of proprioception and discriminatory sensation. Sacral sparing: sparing of tracts to sacral segments with preservation of perianal sensation, rectal sphincter tone, active toe flexion. 65.A patient is recovering from stroke and presents with moderate impairments of the left upper and lower extremities. The PT’s goal today is to instruct the patient in a stand-pivot transfer to the more affected side so the patient can go home on a weekend pass. The spouse is attending today’s session and will be assisting the patient on the weekend. What is the BEST choices for teaching this task? A. Practice the task first with the patient then with the caregiver B. Demonstrate the task, then have the caregiver practice with the patient C. Practice the task first with the caregiver, then with the patient D. Demonstrate the task, and then practice with the patient Correct Answer: D To ensure optimal motor leaning, first demonstrate the task at ideal performance speeds. This provides the patient with an appropriate reference of correction (cognitive map) of the task. Then use guided practice with the patient to ensure safety and successfull performance. Incorrect Choices: Caregivers should become involved only after initial practice of the task with the patient and after the safety of the patient can be assured. 66.A patient presents with low back pain of insidious onset. Based on the history and subjective complaints, the patient appears to have a dysfunction of a lumbar facet joint. What clinical test should be utilized to confirm this diagnosis? A. McKenzie’s side glide test B. Stork standing test C. Slump test D. Lumbar quadrant test Correct Answer: D The motion of the lumbar quadrant test places the lumbar facet joint in its maximally closed and therefore most provocative position, so if positive it is typically indicative of a lumbar facet dysfunction. Incorrect Choices: The slump test is utilized to assess the neurodynamics of the spinal and peripheral nerves. The stork standing test is utilized to identify a spondylolisthesis. McKenzie’s side glide test is utilized to determine if a dise dysfunction with nerve root involvement is present versus a postural disorder. 67.When using a patellar tendon-bearing (PTB) prosthesis, a patient will experience excessive knee flexion in early stance if the: A. Socket is aligned too far back or tilted posteriorly B. Foot position is inset too much C. Socket is aligned too far forward or tilted anteriorly D. Foot position is outset too much Correct Answer: C Ina PTB prosthesis, the socket is normally aligned in slight flexion to enhance loading on the patellar tendon, prevent genu recurvatum and resist the tendency of the amputated limb to slide too deeply into the socket. If it is aligned incorrectly (too far anterior or excessively flexed), it will result in excessive knee flexion. Incorrect Choices: ‘A socket aligned too posterior results in insufficient knee flexion. Excessive foot inset results in lateral thrust at midstance. Excessive foot outset results in medial thrust at midstance. 68.A patient presents with Bluish discoloration of the skin and nail beds of fingers and toes. Palms are also cold and moist. What is the MOST likely cause of these changes? A. Carotenemia B. Hypothyroidism ~. Cyanosis D. Liver disease Correct Answer: C Bluish discoloration of the skin and nailbeds of fingers and toes, along with palms that are cold and moist, is indicative of cyanosis. It is caused by an excess of deoxygenated haemoglobin in the blood. It may be central (due to advanced lung disease, congenital heart disease, abnormal haemoglobin) or peripheral (decreased blood flow, venous obstruction). Incorrect Choices: Liver disease produces jaundice (diffusely yellow skin and sclerae). Carotenemia produces a yellow color, especially in the palms, soles, and face (does not affect the sclerae).. Hypothyroidism produces dry and cool skin. 69.An outpatient physical therapist is examining a patient who underwent a total knee arthroplasty 2 weeks ago. The patient reports that the entire leg has started swelling in the past 2 days. On examination there is pitting edema throughout the lower leg and foot with tendemess throughout the mid calf. Girth measurements reveal a 3.5 cm increase in the size of the mid calf in the symptomatic leg. What recommendation should the physical therapist make to the patient? A. Rest, ice, and elevate the affected lower extremity B. Go home and monitor symptoms. Phone the physician if there is no improvement in 24 hours C. Go immediately to the emergency department D. Go to the physician’s office after the therapy session for further assessment Correct Answer: C The patient scores a 3 on Well’s criteria for deep vein thrombosis, placing himw/herin the high probability category. The most appropriate response for the physical therapist would be to send the patient to the emergency department for further assessment. Incorrect Choices: Any choice except sending the patient to the emergency department would place the patient at an unnecessary risk for developing a pulmonary embolism. 70.A patient complains of persistent wrist pain after painting a house 3 weeks ago. The patient demonstrates signs and symptoms consistent with de Quervain’s tenosynovitis. An appropriate special test to confirm the diagnosis is: A. Finkelstein’s test B. Phalen’s test '. Froment’s sign D. Craig’s test Correct Answer: A Finkelstein’s test is specific for reproducing the pain associated with de Quervain’s tenosynovitis of the abductor pollicis longus and extensor pollicis brevis. Incorrect Choices: Froment’s sign is used to identify ulnar nerve dysfunction. Phalen’s test identifies median nerve compression in the carpal tunnel, Craig’s test identifies an abnormal femoral antetorsion angle, which you hopefully eliminated first. 71.As part of the chart review, the physical therapist views the patient's most current chest film. Based on this film, what is the MOST likely examination finding? A. Increased lateral costal expansion B, Increased subcostal angle C. Decreased inspiration:expiration (I:E) ratio D. Decreased mediate percussion Correct Answer: B This film demonstrates a patient with hyperinflated lungs as evidenced by the flattened diaphragm, blunted costophrenic angle, and increased amount of air. This will cause the subcostal angle to increase significantly. Incorrect Choices: Hyperinflated lungs are indicative of obstructive disease. The I:E ratio will increase in this case as the patient has difficulty getting air out. There is no evidence 72.An elderly patient with persistent balance difficulty and a history of recent falls (two in the past 3 months) is referred for physical therapy examination and evaluation. During the initial examination, what should the therapist examine first? A. Level of dyspnea during functional transfers B. Cardiovascular endurance during a 6-minute walking test C. Sensory losses and sensory organization of balance D. Spinal musculoskeletal changes secondary to degenerative joint disease (DJD) Correct Answer: C Acctitical component of balance control is sensory input from somatosensory, visual and vestibular receptors, and overall sensory organization of inputs. Initial examination should address these elements before moving on to assess the motor components of balance (e.g. postural synergies). The Clinical Test for Sensory Integration in Balance (CTSIB) or modified CTSIB (Shumway-Cook, Horak) are appropriate instruments. Incorrect Choices: Cardiovascular endurance and level of dyspnea during functional transfers are appropriate elements to examine but should occur after key elements of balance are examined (sensory components and integration; motor and synergistic elements). In this case, DJD changes would not be crucial to examine initially. 73.To reduce an elderly individual’s chronic forward head posturing in standing and sitting the therapist should consider stretching exercises to: A. Middle trapezius and rhomboid muscles B. Reetus capitis anterior muscles C. Longus capitis and longus colli muscles D. Reetus capitis posterior minor and rectus capitis posterior major muscles Correct Answer: D Forward head posturing or forward translation of the occiput in relation to the neck and trunk is associated with extension of the occipital axial joint and flexion of the lower and mid cervical spines. Chronic extension of the occipital axial joint will lead to shortening of the suboccipital extensor muscles (rectus capitis posterior major and minor), and localized stretching of these muscles would be indicated as part of a therapeutic intervention to reduce forward head posturing. Incorrect Choices: Muscles anterior to the axis for mid and lower cervical flexion and extension will be chronically overlengthened, and therefore further stretching of these would not be indicated. Forward head posturing is also associated with forward scapular posturing, and therefore further stretching of scapular adductors (middle trapezius and rhomboid muscles) would not be indicated. 74,What are the major benefits of using the 6-Minute Walking Test as an outcome measure? A. Accurately documents maximal exercise capacity B. Provides good correlation with functional abilitites C. Allows determination of severity of lung disease D. Provides determination of peak oxygen uptake Correct Answer: B The 6-Minute Walk Test (6MWT) shows a good correlation with function, as the 6MWT is a submax test, and function is performed at a submax work level. Incorrect Choices: The 6MWT does not correlate to lung disease severity. The 6MWT has only about a 73% correlation with VOsma. The 10-Meter Shuttle Walk Test would be a better test to use if correlation with VO; is desired. As the 6MWT doesn’t correlate with VOsias it cannot document maximal exercise capacity. 75.During a postural screen for a patient complaining of low back pain, the therapist notices that the knees are in genu recurvatum. What are the common contributory problems for which the therapist should examine? A. Ankle dorsiflexion and hip abduction B. Forefoot varus and posterior pelvic tilt ~. Ankle plantarflexion and anterior pelvic tilt D, Lateral tibial torsion and anterior pelvic tilt Correct Answer: C ‘A common contributory problem or correlated motion for genu recurvatum is ankle plantarflexion due to shortened gastrocnemius muscles. Alterations occurring up the kinetic chain include anterior pelvic tilt to maintain the center of gravity over the feet. Incorrect Choices: Ankle dorsiflexion will lead to increased knee flexion. Forefoot varus may lead to tibial internal rotation, but not genu recurvatum. Tibial external rotation will lead to abnormal stresses at the knee joint, but not genu recurvatum. 76.After treating a patient for trochanteric bursitis for 1 week, the patient has no resolution of pain and is complaining of problems with gait. After re-examination, the therapist finds weakness of the quadriceps femoris and altered sensation at the greater trochanter. What is the MOST likely cause of the problems? ‘A. LS nerve root compression B. Sacroiliac (SI) dysfunction C. L4 nerve root compression D. Degenerative joint disease (DJD) of the hip Correct Answer: C The positive findings are consistent with an L4 nerve root compression. Incorrect Choices: ‘Weakness of only one muscle group is not a common finding for DJD or SI dysfunction. LS nerve root compression would result in hamstring weakness 77.A patient in chronic renal failure is being seen in physical therapy for deconditioning and decreased gait endurance. The therapist needs to schedule the patient’s sessions around dialysis, which is received three mornings a week. What guidelines should the therapist follow when taking the patients blood pressure? A. Every minute during walking, using the nonshunt arm B. Pre-and postactivities, using the nonshunt arm C. Insitting when activity has ceased, using the shunt arm D. In the supine position, using the shunt arm Correct Answer: B A dialysis shunt would interfere with taking BP. Using the nonshunt arm, Pre- and postexercise measurements are appropriate. Incorrect Choices: The shunt arm cannot be used to take BP. Taking BP in the shunt arm or during walking would result in inaccurate measurements. 78.A patient who is currently being treated for low back pain arrives for therapy complaining of pain across the middle of the right chest and back. When the therapist inspects the skin, clustered vesicles are apparent in a linear are, The surrounding skin is hypersensitive. What is the MOST likely diagnosis? ‘A. Herpes simplex infection B. Psoriasis C. Dermatitis D. Herpes zoster infection Correct Answer: D Herpes zoster is an acute infection caused by reactivation of the latent varicella-zoster virus (shingles). It is characterized by painful vesicular skin eruptions that follow the underlying route of a spinal (in this case) or cranial nerve. Additional symptoms include fever, gastrointestinal disturbances, malaise, and headache. Incorrect Choices: Herpes simplex is an infection caused by the herpes simplex virus. These infections tend to occur on the face (around the mouth and nose). They are sometimes referred to as “cold sores.” Psoriasis is a chronic skin condition characterized by ted patches covered by dry, silvery scales. Dermatitis is an inflammatory condition of the skin characterized by eruptions (not associated with an underlying route of a nerve). 79.While providing sports coverage at a local high school, a physical therapist is asked to examine an athlete with a knee injury. Based on the mechanism of injury, the therapist suspects rupture of the ACL. What test should be performed immediately to identify a to ACL? ‘A. MeMurray’s test B. Reverse Lachman’ stress test C. Lachman stress test D. Posterior sag test Correct Answer: C Lachman’s stress test is the primary clinical test utilized to identity if the anterior cruciate ligament is intact or ruptured. Incorrect Choices: McMurray’s test is utilized to identify the integrity for the meniscus. The reverse Lachman’s, stress and posterior sag tests are both tests utilized to test the integrity for the meniscus. The reverse Lachman’s stress and posterior sag tests are both tests utilized to test the integrity of the posterior cruciate ligament. 80.After a myocardial infarction (MI), a patient is a new admission to a phase 3 hospital-based cardiac rehabilitation program. During the initial exercise session, the patient's ECG responses are continuously monitored via radio telemetry. The therapist notices three PVCs occurring in a run with no P wave. The therapist should: A. Modify the exercise prescription by decreasing the intensity B. Stop the exercise and notify the physician immediately C. Continue the exercise session, but monitor closely D. Have the patient sit down and rest for a few minutes before resuming exercise Correct Answer: B Arun of three or more PVCs occurring sequentially is ventricular tachycardia. The rate is very rapid, resulting in seriously compromised cardiac output. This is potentially an emergency situation that can deteriorate rapidly into ventricular fibrillation (no cardiac output) and cardiac arrest. Incorrect Choices: The other choices, which involve continuation of exercise, put the patient at serious risk for cardiac arrest. 81.A patient with left hemiplegia is able to recognize his wife after shie is with him for a while and talks to him, but is unable to recognize the faces of his children when they come to visit. The children are naturally very upset by their father’s behavior. The BEST explanation for his problem is: A. Somatognosia B. Anosognosia C. Visual agnosia D. ideational apraxia Correct Answer: C All of the choices are indicative of perceptual dysfunction. This patient is most likely suffering from visual agnosia, which is an inability to recognize familiar objects despite normal function of the eyes and optic tracts. Once the wefe talks with him, he is able to recognize her by her voice. Incorrect Choices: Ideational apraxia is the inability to perform a purposeful motor act, either automatically or upon command. Anosognosia is the frank denial, neglect, or lack of awareness of the presence or severity of one’s paralysis. Somatognosia is an impairment in body scheme. 82.A home care PT receives a referral to evaluate the fall risk potential of an elderly community-dweller with chronic coronary artery disease (CAD). The patient has fallen three times in the past 4 months, with no history of fall injury except for minor bruising. The patient is currently taking a number of medications. What is the drug that is MOST likely to contribute to dizziness and increased fall risk? A. Colace B. Albuterol C. Nitroglycerin D. Coumadin sodium Correct Answer: C Of the medications listed, nitroglycerin has the greatest risk of causing dizziness or weakness due to postural hypotension. Fall risk is increased even with small doses of nitroglycerin. Incorrect Choices: Colace (docusate sodium), an anticonstipation agent, can result in mild abdominal cramps and nausea. Coumadin (warfarin sodium) is an anticlotting medication. Adverse effects can include increased risk of haemorthage, which indirectly can result in lightheadedness. Dosages are carefully monitored. Albutero, a bronchodilator, can cause tremor, anxiety, nervousness, and weakness. 83.A patient complains of foot pain when first arising that eases with ambulation. The therapist finds that symptoms can be reproduced in weight bearing and running on a treadmill. Examination reveals pes planus and pain with palpation at the distal aspect of the calcaneus. Early management would include: A. Prescription for a customized orthosis B. Strengthening of ankle dorsiflexors C. Modalities to reduce pain D, Use of a resting splint at night Correct Answer: D The symptoms are suggestive of plantar fasciitis. The focus of patient management should be on decreasing the irritation to the plantar fascia. This is most effectively done with a resting night splint. Incorrect Choices: Modalities to reduce pain offer some symptomatic relief, however, the pain is not constant. Strengthening the dorsiflexors will not change irritation to the plantar fascia. A customized orthosis may be necessary at a later time if primary symptoms do not resolve after early management. 84.A patient with a T10 paraplegia (ASIA A) resulting froma spinal cord injury is ready to begin community wheelchair training. The therapist's goal is to teach the patient how to do a wheelie in order to manage curbs. What is the BEST training strategy to instruct the patient in performing a wheelie? A. Place a hand on the top of the handrims to steady the chair while throwing the head and trunk forward. B. Throw the head and trunk backward to rise up on the large wheels C. Lean backward while moving the hands slowly backward on the rims D. Grasp the handrims posteriorly, and pull them forward abruptly and forcefully Correct Answer: D A wheelie can be assumed by having the patient place his/her hands posterior on the handrims and pulling them abruptly and sharply forward. If the patient is unable to lift the casters in this manner, throwing the head back forcefully when pulling the handrims may work. An alternate technique is to grasp the handrims anteriorly, pull backward, then abruptly and forcefully reverse the direction of pull. The therapist can assist by steadying the chair at the patient's balance point until the patient learns to adjust the position through the use of handrim. movements forward and backward. Incorrect Choices: ‘Throwing the head and trunk backward alone or with moving the hands backward on the handrims will not result in a wheelie. 85.A PT should be alert to recognize the signs and symptoms associated with the onset of aspiration pneumonia, Which patient diagnosis is the MOST susceptible to develop this from of pneumonia? A. A circumferential bum of the thorax associated with significant pain B. Severe scoliosis with compression of internal organs, including the lungs CC. Amyotrophic lateral sclerosis (ALS) with dysphagia and diminished gag reflex D. A complete spinal cord lesion at T2 with diminished coughing ability and forced vital capacity (FVC) Correct Answer: C Aspiration pneumonia results from an abnormal entry of fluids or matter (including food) into the airways. A patient with ALS with an inability to swallow (dysphagia) and diminished gag reflex is most susceptible to aspiration pneumonia. Incorrect Choices: Others listed may be susceptible to other forms of pneumonia or even, though less likely, aspiration pneumonia. 86.A patient demonstrates quadriceps weakness (4/5) and difficulty descending stairs. The BEST intervention to regain functional strength in the quadriceps A. Progressive resistance exercises, 70% 1 repetition maximum, three sets of 10 B. Partial squats, progressing to lunges CC. Maximum isometric exercise, at 45° and 90° of knee extension D. Isokinetic exercise, at 36%see Correct Answer: B Closed-chain exercises are the most appropriate in this example because of the patient's difficulty descending stairs. Moving the body over a fixed distal segment provides loading to muscles, joints, and noncontractile soft tissues while stimulating the sensory receptors needed for stability and balance. Incorrect Choices: Open-chain exercise (all other choices), while improving strength, does not adequately prepare an individual for functional weight bearing activities. 87.Independent community ambulation as the primary means of functional mobility is a realistic functional expectation for a patient with the highest level of complete spinal cord injury (ASIA A) at: A. Low lumbar (L4-5) B. Low thoracic (T9-10) C. Midthoracic (T6-9) D. High lumbar (T12-L1) Correct Answer: A Patients with low lumbar lesions (L4-5) can become independent and functional ambulators providing that they exhibit adequate ROME strength of residual muscles and cardiovascular endurance. The locomotor training program emphasizes learning to use bilateral ankle-foot orthoses (AFOs) and canes. Incorrect Choices: Patients with complete higher lesions (all other choices) can leam to ambulate with KAFOs and crutches, but may demonsirate limited function and a high rate of orthotic rejection due to the high levels of energy expenditure required during ambulation. These patients typically choose wheelchair mobility as their primary means of locomotion. 88.A patient presents with multiple fractures of both hands and wrists as a result of a mountain bike accident. Now, 5 weeks later, the patient has limited wrist and finger motion and dry. scaly skin over the involved areas, What is the biophysical agent that would provide the GREATEST benefit? A. Contact ultrasound (US) B. Hot packs C. Paraffin D. Functional electrical stimulation Correct Answer: C Paraffin bath will provide circumferential heating of the hands and fingers and aid in softening the skin prior to exercise. Incorrect Choices: Active exercise, including functional electrical stimulation, would be more effective after the application of paraffin because tissue extensibility and pliability would be increased. Hot packs or US using direct contact would not completely cover the area to be treated. 89.An elderly patient demonstrates a history of recent falls (two in the past 2 months) and mild balance instability. The therapist’s referral is to examine the patient and recommended an. assistive device as needed. Based on the patient's history, it would be BEST to select a: A. Folding reciprocal walker B. Standard, fixed-frame walker C. Front wheel rolling walker that folds D. Hemi walker Correct Answer: C A rolling walker will provide added stability, while maintaining gait as a continuous movement sequence. The additional benefit of a folding walker facilitates easy transport and mobility in the community. Incorrect Choices: ‘A standard fixed-frame walker requires the patient to lift the walker and increases the energy expenditure. A hemi walker decreases the stability offered. A reciprocal walker requires increased motor control to use and also provides i 90.After surgery, a patient develops a stiff pelvis and limited pelvic/lower trunk mobility. The therapist elects to use sitting exercises on a therapy ball to correct these impairments. In order to improve lower abdominal control, what direction should the patient move the ball? A. Backward, producing anterior tilting of the pelvis, B. Forward, producing posterior tilting of the pelvis C. Forward, producing anterior tilting of the pelvis, D. Backward, producing posterior tilting of the pelvis Correct Answer: B Contraction of the lower abdominals results in posterior tilting of the pelvis and can be achieved with forward or anterior movement of the therapy ball. Incorrect Choices: The other choices incorrectly state the effects of pelvic control exercises while sitting on the ball. Backward or posterior motion of the ball produces anterior tilting of the pelvis. 91.A patient presents with weakness in the right lower leg 3 weeks after a motor vehicle accident. The patient complains of spontaneous twitching in the muscles of the lower leg. The therapist visually inspects both limbs and determines that muscle bulk is reduced on the involved right limb. Girth measurements confirm a I-inch difference in the circumference of the right leg measured 4 inches below the patella. Deep tendon reflexes and tone are diminished. What is the MOST likely cause of the patient’s weakness? A. Peripheral nerve injury B. Pyramidal tract dysfunction in the medulla ~ Guillan-Barre syndrome D. Brainstem dysfunction affecting extrapyramidal pathways Correct Answer: A This patient is exhibiting signs and symptoms of lower motor neuron injury (hypotonia, hyporeflexia, paresis, neurogenic atrophy). The presence of muscle fasciculations is a hallmark sign of lower motor injury. Incorrect Choices: Upper motor neuron lesions (cortical or pyramidal tracts) would result in hypertonicity (hypotonicity initially during shock), hyperreflexia, generalized paresis, and variable disuse atrophy. Guillain-Barre syndrome is a lower motor neuron condition that produces symmetrical and ascending signs. Extrapyramidal signs (involuntary movements) are not evident in this case. 92.An elderly adult patient presents with a history of and subjective complaints consistent with lumbar central spinal stenosis. The most appropriate clinical test used to differentiate spinal stenosis from intermittent vascular claudication would be: A. Femoral nerve traction test B. Bicycle (van Gelderen’s) test C. Valsalva’s maneuver D. Lumbar quadrant test Correct Answer: B The bicycle (van Gelderen’s) test is designed to differentiate between spinal stenosis and intermittent vascular claudication. Van Gelderen’s bicycle test is designed to stress the LE vascular system without causing any central canal or foraminal stenosis that could be misinterpreted as intermittent neurogenic claudication. Incorrect Choices: The lumbar quadrant test is utilized to identify a lumbar facet dysfunction. The femoral nerve traction test is utilized to identify if there is an entrapment of the femoral nerve and thus it is not related to either of these two conditions directly. Valsalva’s maneuver is utilized to identify a space-occupying lesion such as a dise herniation, a tumor, ete. this would not be sensitive in identifying a spinal stenosis. 93.A patient with bacterial pneumonia has crackles and wheezes in the left lateral basal segment and decreased breath sounds throughout. The patient is on 4 L of oxygen by nasal cannula with a resulting arterial oxygen saturation (SaO2) of 90%. Respiratory rate is 28. What is the MOST BENFFICIAL intervention for this case? A. Postural drainage, percussion, and shaking over the appropriate area on the left lateral thorax for secretion removal B. Positioning in left side-lying to improve ventilation/perfusion ratios C. Postural drainage, percussion, and shaking to the right basilar segments in order to keep the right lung healthy D. Breathing exercise encouraging expansion of the right lateral basilar thorax, because the left side is not currently participating in gas exchange Correct Answer: A A treatment of postural drainage, percussion, and shaking to the appropriate lung segments is, advisable. The standard postural drainage position for the lateral basilar segment of the left lower lobe is in side-lying position with the head of bed tipped in full Trendelenburg position. Given the borderline SaO; values on 4 L of oxygen, modification of the position may be necessary for patient tolerance. Incorrect Choices: There is nothing that will ensure that the right hung stays healthy or makes it more functional. Therefore, positioning in left side-lying and focusing on left-sided breathing exercises are incorrect choices. In addition, placing the patient in the side-Lying position would increase blood flow to the left lateral base, an area that is getting little ventilation. This position would worsen ventilation/perfusion matching. 94.4 therapist receives a referral to see an elderly patient in the intensive care unit (ICU) recovering from a severe case of pneumonia. The therapist recognizes that the disorientation is due to delirium rather than dementia because the: A. Patient has hallucinations throughout the day B. Patient demonstrates persistent personality changes C. Symptoms are intermittent D. Level of arousal is significantly depressed Correct Answer: C Acutely ill, hospitalized elderly patients frequently exhibit delirium, a fluctuating attention state. Patients demonstrate a fluctuating course with symptoms of confusion that alternate with lucid intervals. Sleep/wake cycles are disrupted and confusion is typically worse at night. Incorrect Choices: Allother choices are signs of chronic dementia. 95.A comparison of the effects of exercise in water, on land, or combined on the rehabilitation outcome of patients with intra-articular anterior cruciate ligament reconstructions revealed that less joint effusion was noted after 8 weeks in the water group. An appropriate statistical test to compare the girth measurements of the three groups is: A. Analysis of covariance B. Spearman's rho C. Chi-square D. ANOVA Correct Answer: D ANOVA is a parametric statistical test used to compare three or more treatment groups (in this example, in water, on land, or combined exercise groups) on a measure of the dependent variable (joint effusion girth measurements) at a selected probability level. Incorrect Choices: ‘Analysis of covariance compares two or more groups but also controls for the effects of an intervening variable. Chi square is a nonparametric statistical test used to compare data in the form of frequency counts. Spearman’s rho (rank correlation coefficient) is a nonparametric test used to correlate ordinal data. 96.A patient has a recent history of strokes (two in the past 4 months) and demonstrates good return in the right lower extremity. The therapist is concentrating on improving balance and independence in gait. Unfortunately, speech recovery is lagging behind motor recovery. The patient demonstrates a severe fluent aphasia. What is the BEST strategy to use during physical therapy sessions? ‘A. Demonstrate and gesture to get the idea of the task across B. Have the family present to help interpret during physical therapy sessions C. Utilized verbal cues, emphasizing consistency and repetition D. Consult with the speech pathologist to establish a communication board Correct Answer: A Fluent aphasia (Wemicke’s aphasia) is a central language disorder in which spontaneous speech is preserved and flows smoothly while auditory comprehension is impaired. Demonstration and gesture (visual modalities) offer the best means of communicating with this patient since the impairment is severe. Incorrect Choices: Verbal cues are best for patient with nonfluent aphasia (broca’s aphasia) in which understanding of verbal cues Is intact but motor production of speech is not. A communication board is not needed. The family will also have difficulties communicating with patient. Use of consistency and repetition is an appropriate motor learning strategy for all patients. 97.During a gait examination, a patient with a transfemoral prosthesis demonstrates terminal swing impact. Additional examination is needed as the therapist suspeets the: A. Prosthesis has too little tension in the extension aid B. Hip flexors are weak C. Prosthesis is externally rotated D. Prosthesis has insufficient knee friction Correct Answer: D Terminal swing impact refers to the sudden stopping of the prosthesis as the knee extends during late swing. Possible causes can include insufficient knee friction or too much tension in the extension aid. In addition, if the patient with an amputation fears the knee will buckle at heel strike, the patient can use forceful hip flexion to extend the knee. Incorrect Choices: Faulty socket contour results in an externally rotated prosthesis. The extension aid has too much tension (not too little). Weak hip flexors would result in decreased force to extend the knee. 98.A patient was burned over 40% of the body in an industrial accident and has full-thickness bums over the anterior trunk and neck and superficial partial-thickness bums over the shoulders. In order to stabilize this patient out of positions of common deformity, what orthotic device would be of GREATEST benefit? A. Soft cervical collar with an intrinsic plus hand splint B. A cervical thoracic lumbosacral orthosis (CTLSO) used during all upright activities C. Plastic cervical orthosis and axillary splints utilizing an airplane position D. Splints utilizing a flexed position for the shoulders and body jacket for the trunk Correct Answer: C The common deformity for the anterior neck is flexion; the appropriate positioning device is a firm rigid plastic cervical collar that stresses extension. The common deformity of the shoulders in adduction and internal rotation; the appropriate position device is an axillary or airplane splint that stresses abduction, flexion, and external rotation. Incorrect Choices: Choices that involve hand splints should be ruled out immediately because there is no mention of burns to the hands. A CTLSO would prevent neck flexion but does not deal with the potential shoulder deformities. The CTLSO could also restrict breathing and enhance the risk of pneumonia. 99. patient with type 2 diabetes is referred to physical therapy for exercise conditioning. What isa pathophysiologic cause of type 2 diabetes? A. Metabolic syndrome B. Impaired ability of the tissues to use insulin and insulin deficiency C. Loss of beta-cell function and insulin deficiency D. Pancreatic tumor Correct Answer: B Type 2 diabetes results from impaired ability of the tissues to use insulin (insulin resistance), accompanied by a relative lack of insulin or impaired release of insulin. Incorrect Choices: Type 1 diabetes results from loss of pancreatic beta-cell function and an absolute insulin deficiency. Metabolic syndrome Is a precursor to type 2 diabetes and is evidenced by abdominal obesity, high triglycerides, low high-density lipoprotein (HDL), hypertension, and high fasting plasma glucose (> 110 mg/dL). Pancreatic tumor is not a causative factor in type 2 diabetes. 100. Which intervention is BEST to improve left-sided neglect in a patient with left hemiplegia? A. Hook-lying, holding, light resistance to both hip abductors B. Rolling, supine to side-lying on right, using a PNF lift pattem C. Sitting, with both arms extended, hands resting on support surface, active holding D. Bridging with both arms positioned in extension at the sides Correct Answer: B Incorporating the involved left side into a crossing the midline activity (rolling, using PNF lift) is best. Incorrect Choices: All other choices involve symmetrical activity and do little to bring attention to the involved hemiplegic side. 101. A patient with no significant past medical history who now presents with a bacterial pneumonia in the right anterior base would present with which of the following exam findings? A. Decreased breath sounds throughout all lung fields, increased SaO>, febrile B. Bronchial breath sounds at the right anterior base, increased SaQ>, febrile C. Crackles on inspiration only at right anterior base, decreased SaO, and productive cough x3 days D. Wheezes on inspiration only throughout the right lung fields, decreased SaO,, dry cough x1 day. Correct Answer: C Bacterial pneumonia has a gradual onset of days with a productive cough. As pneumonia interferes with the transport of oxygen from the alveoli to the pulmonary capillaries, the PaO and therefore the SaO would be lower than expected, and crackles in the area of the pneumonia is a usual finding. Incorrect Choices: Wheezing may occur with a bacterial pneumonia and resound within that thorax, and a decrease in SaO2, making this portion of the answer plausible, But a more abrupt onset with a dry cough is consistent with a viral infection, not bacterial infection. An increased SaQz would not be seen, negating this answer as a possibility. Febrile (with fever) certainly would be plausible, but the overall decreased breath sounds don’t fit with pneumonia in a specific region of the lung. An increased SaO, would not be seen, negating this answer as a possibility. Febrile (with fever) certainly would be plausible, as is a bronchial breath sound in the specific involved region of the lung. 102. A patient with chronic obstructive pulmonary disease (COPD) reports to the fourth outpatient pulmonary rehabilitation session complaining of nausea, gastric upset, and feeling jittery. The patient reports no change in pulmonary symptoms. The PT records the following set of vital signs: temperature 98.6°F. HR. 110 beats/min and irregular, BP 150/86, respiratory rate 20. Breath sounds show no change from baseline. The therapist checks the medical record and finds that the patient has no history of gastric disease. The patient is presently taking theophylline, albuterol sulfate (ventolin), and triamcinolone diacetate (Amcort). What action should the PT take? A. Call the patient’s physician immediately and report signs of theophylline toxicity B. Have the patient increase use of Ventolin to improve respiratory status C. Have the patient stop use of Amcort until he/she schedules an appointment with the physician D. Send the patient home and notify the physician of current symptoms Correct Answer: A Theophylline is a bronchodilator used to reverse airway obstruction. The combination of symptoms of imegular HR, feeling jittery, and gastric upset is consistent with theophylline toxicity, Because theophylline toxicity can cause arrhythmias and seizures, the patient's physician should be notified by the PT rather than wait for the patient to retum home to call the physician, It is also likely that a blood test will be needed to check the theophylline toxicity level and this could be done at the facility. Incorrect Choices: Ventolin is a bronchodilator used in the treatment of asthma or COPD. Amcort is an anti- inflammatory agent used to manage bronchial asthma, Neither drug produces the same combination of symptoms described in the case. The therapist should not recommend increasing or stopping a prescribed medication. This is usually the physician’s responsibility. 103. The therapist is treating a 1-year-old child with Down syndrome at home and notices decreasing strength in the extremities, with neck pain and limited neck motion. Upper extremity deep tendon reflexes (DTRs) are 3+. These signs and symptoms are a hallmark of what diagnosis? A. Lower motor neuron signs consistent with Down syndrome B. Atlanto-axial subluxation with lemniscal impingement C. Upper motor neuron signs consistent with Down syndrome D. Atlanto-axial subluxation with spinal cord impingement Correct Answer: D Ligamentous laxity is a hallmark of Down syndrome and can lead to atlanto-axial instability (AAI) with spinal cord impingement. This is a medical emergency situation. Decreased muscle strength and increased DTRs are the signs of dislocation from loss of cord function. In this case,

You might also like